Sie sind auf Seite 1von 39

EE 369

POWER SYSTEM ANALYSIS


Lecture 16
Economic Dispatch
Tom Overbye and Ross Baldick

Announcements
Read Chapters 6 (section 6.12) and 7
(sections 7.1 to 7.3).
Homework 12 is 6.62, 6.63, 6.67
(calculate economic dispatch for values
of load from 55 MW to 350 MW); due
Tuesday, 11/29.
Class review and course evaluation on
Tuesday, 11/29.
Midterm III on Thursday, 12/1, including
material through Homework 12.
2

Economic Dispatch:
Formulation
The goal of economic dispatch is to determine
the generation dispatch that minimizes the
instantaneous operating cost, subject to the
constraint that total generation = total load +
losses

Minimize

CT @ Ci ( PGi )
i 1

Such that
m

PGi PD PLosses
i 1

Initially we'll
ignore generato
limits and the
losses
3

Unconstrained Minimization
This is a minimization problem with a single
equality constraint
For an unconstrained minimization a
necessary (but not sufficient) condition for a
minimum is the gradient of the function must
be zero,
The gradient generalizes
f (x) 0 the first derivative
for multi-variable problems:

f (x) f (x)
f (x)
f (x) @
,
,K ,

1
2
n
4

Minimization with Equality


Constraint
When the minimization is constrained with an
equality constraint we can solve the problem using
the method of Lagrange Multipliers
Key idea is to represent a constrained minimization
problem as an unconstrained problem.

That is, for the general problem


minimize f (x) s.t. g(x) 0
We define the Lagrangian L(x, ) f (x ) Tg (x )
Then a necessary condition for a minimum is the
L x(x, ) 0 and L (x , ) 0

Economic Dispatch
Lagrangian

For the economic dispatch we have a minimization


constrained with a single equality constraint
L(PG , )

i 1

i 1

Ci ( PGi ) ( PD PGi )

(no losses)

The necessary conditions for a minimum are


L(PG , )
dCi ( PGi )

0 (for i 1 to m)
PGi
dPGi
m

PD PGi 0
i 1

Economic Dispatch Example


What is economic dispatch for a two generator
system PD PG1 PG 2 500 MW and
C1 ( PG1 ) 1000 20 PG1 0.01PG21

$/h

C2 ( PG 2 ) 400 15PG 2 0.03PG22

$/h

Using the Lagrange multiplier method we know:


dC1 ( PG1 )

20 0.02 PG1
0
dPG1
dC2 ( PG 2 )

dPG 2

15 0.06 PG 2

500 PG1 PG 2 0
7

Economic Dispatch
Example, contd
We therefore need to solve three linear equations
20 0.02 PG1
0
15 0.06 PG 2

500 PG1 PG 2 0
0
1 P G1
0.02
20
0
0.06 1 P G 2 15

1 0
500
1

PG1
312.5 MW
P 187.5 MW
G 2


26.2 $/MWh

Economic dispatch example,


contd
At the solution, both
generators have the same
marginal (or incremental) cost, and this
common marginal cost is equal to .
Intuition behind solution:
If marginal costs of generators were different, then
by decreasing production at higher marginal cost
generator, and increasing production at lower
marginal cost generator we could lower overall costs.
Generalizes to any number of generators.

If demand changes, then change in total costs


can be estimated from .

Economic dispatch example,


contd
Another way to solve the equations is to:
Rearrange the first two equations to solve for PG1
and PG2 in terms of ,
Plug into third equation and solve for ,
Use the solved value of to evaluate PG1 and PG2.

This works even when relationship between


generation levels and is more complicated:
Equations are more complicated than linear when
there are maximum and minimum generation
limits or we consider losses.
10

Lambda-Iteration Solution
Method
Discussion on previous page leads to lambdaiteration method:
this method requires a unique mapping from a value
of lambda (marginal cost) to each generators MW
output:
for any choice
PGi ( ).of lambda (common marginal cost),
the generators collectively produce a total MW
output,
the method then starts with values of lambda below
and above the optimal value (corresponding to too
little and too much total output), and then iteratively
brackets the optimal value.

11

Lambda-Iteration Algorithm
Pick L and H such that
m

L
P
(

Gi ) PD 0
i 1

H
P
(

Gi ) PD 0
i 1

H L Do

While

M ( H L ) / 2
If

M
H
M
P
(

0
Then

Gi
D
i 1

Else L M
End While

12

Lambda-Iteration: Graphical
View

In the graph shown below for each value of lam


there is a unique PGi foreachgenerator.This
relationshipisthePGi()function.

13

Lambda-Iteration Example
Consider a three generator system with
IC1 ( PG1 ) 15 0.02 PG1
$/MWh
IC2 ( PG 2 ) 20 0.01PG 2

$/MWh

IC3 ( PG 3 ) 18 0.025 PG 3

$/MWh

and with constraint PG1 PG 2 PG 3 1000MW


Rewriting generation as a function of , PGi ( ),
we have

15
PG1 ( )
0.02
18
PG3 ( )
0.025

20
PG2 ( )
0.01
14

Lambda-Iteration Example,
contd
m
L
L
Pick so

PGi (

) 1000 0 and

i=1
m

H
P
(

Gi ) 1000 0
i=1

Try

20 then

PGi (20) 1000

i 1

15 20 18

1000 670 MW
0.02
0.01 0.025
Try H 30 then

PGi (30) 1000

1230 MW

i 1

15

Lambda-Iteration Example,
contd
Pick convergence tolerance 0.05 $/MWh

Then iterate since H L 0.05

M ( H L ) / 2 25
Then since

H
P
(25)

1000

280
we
set

25
Gi
i 1

Since 25 20 0.05

M (25 20) / 2 22.5


m

L
P
(22.5)

1000

195
we
set

22.5
Gi
i 1

16

Lambda-Iteration Example,
contd
Continue iterating until H L 0.05
*

The solution value of , , is 23.53 $/MWh


Once * is known we can calculate the PGi
23.53 15
PG1 (23.5)
426 MW
0.02
23.53 20
PG 2 (23.5)
353 MW
0.01
23.53 18
PG 3 (23.5)
221 MW
0.025
17

Thirty Bus ED Example

Case is economically dispatched (without con


the incremental impact of the system losses).

18

Generator MW Limits
Generators have limits on the minimum
and maximum amount of power they can
produce
Typically the minimum limit is not zero.
Because of varying system economics
usually many generators in a system are
operated at their maximum MW limits:
Baseload generators are at their maximum
limits except during the off-peak.

19

Lambda-Iteration with Gen


Limits
In the lambda-iteration method the limits are taken
into account when calculating PGi ( ) :
if calculated production for PGi PGi ,max
then set PGi ( ) PGi ,max
if calculated production for PGi PGi ,min
then set PGi ( ) PGi ,min
20

Lambda-Iteration Gen Limit


In the previous threeExample
generator example assume
the same cost characteristics but also with limits
0 PG1 300 MW 100 PG2 500 MW
200 PG3 600 MW
With limits we get:
m

PGi (20) 1000


i 1

PG1 (20) PG 2 (20) PG 3 (20) 1000

250 100 200 1000


450 MW (compared to 670MW)
m

PGi (30) 1000


i 1

300 500 480 1000 280 MW


21

Lambda-Iteration Limit
Example,contd
Again we continue iterating until the convergence
condition is satisfied.
With limits the final solution of , is 24.43 $/MWh
(compared to 23.53 $/MWh without limits).
Maximum limits will always cause to either increase
or remain the same.
Final solution is:
PG1 (24.43) 300 MW (at maximum limit)
PG 2 (24.43) 443 MW
PG 3 (24.43) 257 MW
22

Back of Envelope Values


$/MWhr = fuelcost * heatrate + variable O&M

Typical incremental costs can be roughly


approximated:
Typical heatrate for a coal plant is 10, modern
combustion turbine is 10, combined cycle plant is 6 to
8, older combustion turbine 15.
Fuel costs ($/MBtu) are quite variable, with current
values around 2 for coal, 3 to 5 for natural gas, 0.5 for
nuclear, probably 10 for fuel oil.
Hydro costs tend to be quite low, but are fuel (water)
constrained
Wind and solar costs are zero.

23

Inclusion of Transmission
Losses
The losses on the transmission system are a
function of the generation dispatch.
In general, using generators closer to the
load results in lower losses
This impact on losses should be included
when doing the economic dispatch
Losses can be included by slightly rewriting
the Lagrangian to include losses PL:

L(PG , )

Ci ( PGi )

i 1

PD PL ( PG ) PGi

i 1

24

Impact of Transmission
Losses

The inclusion of losses then impacts the necessary


conditions for an optimal economic dispatch:

PD PL ( PG ) PGi .

i 1
i 1
The necessary conditions for a minimum are now:
L(PG , )

Ci ( PGi )

L(PG , )
PGi

dCi ( PGi )
PL ( PG )

1
0

dPGi
PGi

PD PL ( PG ) PGi 0
i 1

25

Impact of Transmission
Losses

dCi ( PGi )
PL ( PG )
Solving for , we get:
1
0

dPGi
PGi

dCi ( PGi )
1

dPGi

PL ( PG )
1 P

Gi
Define the penalty factor Li for the i

th

generator

(don't confuse with Lagrangian L!!!)


1
The penalty factor
Li
at the slack bus is

PL ( PG )
1 P

always unity!

Gi
26

Impact of Transmission
Losses
The condition for optimal dispatch with losses is then
L1 IC1 ( PG1 ) L2 IC2 ( PG 2 ) Lm ICm ( PGm )
1
Li
. So, if increasing PGi increases

PL ( PG )
1 P

Gi
PL ( PG )
the losses then
0 Li 1.0
PGi
This makes generator i appear to be more expensive
(i.e., it is penalized). Likewise Li 1.0 makes a generator
appear less expensive.

27

Calculation of Penalty
Factors

Unfortunately, the analytic calculation of Li is

somewhat involved. The problem is a small change


in the generation at PGi impacts the flows and hence
the losses throughout the entire system. However,
using a power flow you can approximate this function
by making a small change to PGi and then seeing how
the losses change:
PL ( PG ) PL ( PG )

PGi
PGi

1
Li
PL ( PG )
1
PGi

28

Two Bus Penalty Factor


Example

PL ( PG )
0.0387
PG 2

PL ( PG ) 0.37 MW

0.037
PG 2
10 MW

L2 0.9627

L2 0.9643

29

Thirty Bus ED Example

Now consider losses.


Because of the penalty factors the generator in
costs are no longer identical.

30

Area Supply Curve

The area supply curve shows the cost to prod


next MW of electricity, assuming area is econo
dispatched10.00
7.50

Supply
curve for
thirty bus
system

5.00

2.50

0.00
0

100

200
Total Area Generation (MW)

300

400

31

Economic Dispatch Summary


Economic dispatch determines the best way to
minimize the current generator operating costs.
The lambda-iteration method is a good approach
for solving the economic dispatch problem:
generator limits are easily handled,
penalty factors are used to consider the impact of
losses.

Economic dispatch is not concerned with


determining which units to turn on/off (this is the
unit commitment problem).
Basic form of economic dispatch ignores the
transmission system limitations.
32

Security Constrained ED
or Optimal Power Flow
Transmission constraints often limit
ability to use lower cost power.
Such limits require deviations from
what would otherwise be minimum
cost dispatch in order to maintain
system security.
Need to solve or approximate power
flow in order to consider transmission
constraints.
33

Security Constrained ED
or Optimal Power Flow

The goal of a security constrained ED or


optimal power flow (OPF) is to determine the
best way to instantaneously operate a power
system, considering transmission limits.
Usually best = minimizing operating cost,
while keeping flows on transmission below
limits.
In three bus case the generation at bus 3 must
be limited to avoid overloading the line from
bus 3 to bus 2.

34

Security Constrained
Dispatch
Bus 2

-22 MW
4 MVR

22 MW
-4 MVR

Bus 1
1.00 PU

357 MW
179 MVR

1.00 PU
0 MW
37 MVR

100%
194 MW OFF AGC -142 MW
49 MVR
232 MVR AVR ON
145 MW 100%
-37 MVR

Home Area

Bus 3

Scheduled Transactions
100.0 MW

-122 MW
41 MVR

100 MW

124 MW
-33 MVR
1.00 PU
179 MW
89 MVR

448 MW AGC ON
19 MVR AVR ON

Need to dispatch to keep line


from bus 3 to bus 2 from overloading
35

Multi-Area Operation
In multi-area system, rules have been
established regarding transactions on tie-lines:
In Eastern interconnection, in principle, up to
nominal thermal interconnection capacity,
In Western interconnection there are more
complicated rules

The actual power that flows through the entire


network depends on the impedance of the
transmission lines, and ultimately determine
what are acceptable patterns of dispatch:
Can result in need to curtail transactions that
otherwise satisfy rules.

Economically uncompensated flow through


other areas is known as parallel path or loop
flows.
Since ERCOT is one area, all of the flows on AC
lines are inside ERCOT and there is no
36
uncompensated flow on AC lines.

Seven Bus Case: One-line


System has
three areas

44 MW

-42 MW
-31 MW
0.99 PU
3

1.05 PU
1

106 MW -37 MW
AGC ON

62 MW

79 MW

2
40 MW
20 MVR

Top Area Cost


8029 $/MWH

1.00 PU

-32 MW

Case Hourly Cost


16933 $/MWH
32 MW

80 MW
30 MVR

110 MW
40 MVR

38 MW
-61 MW
1.04 PU

31 MW

-77 MW
5
-39 MW

40 MW

94 MW
AGC ON
-14 MW
1.01 PU
130 MW
40 MVR

168 MW AGC ON
-40 MW

20 MW

-20 MW

1.04 PU
6

Left area
has one
bus

40 MW
1.04 PU

20 MW

200 MW
Left Area Cost
0 MVR
4189 $/MWH
200 MW AGC ON

-20 MW

Top area
has five
buses

No net
interchange
between
Any areas.

200 MW
Right Area Cost
0 MVR
4715 $/MWH
201 MW AGC ON

Right area has one


bus

37

Seven Bus Case: Area View


Top
40.1 MW
0.0 MW

Area Losses
7.09 MW

-40.1 MW
0.0 MW

System has
40 MW of
Loop Flow
Left
Area Losses
0.33 MW

Right
40.1 MW
0.0 MW

Actual
flow
between
areas

Scheduled
flow

Area Losses
0.65 MW

Loop flow can result in higher losses


38

Seven Bus - Loop Flow?


Top
4.8 MW
0.0 MW

-4.8 MW
0.0 MW

Left
Area Losses
-0.00 MW

Area Losses
9.44 MW

Right
104.8 MW
100.0 MW

100 MW Transaction
between Left and Right

Note that
Tops
Losses hav
increased
from
7.09MW to
9.44 MW

Area Losses
4.34 MW

Transaction has
actually
39
decreased

Das könnte Ihnen auch gefallen